You are on page 1of 7

Algebra class ex

Q In a 4- digit number, the sum of the first two digits is equal to that of the last two digits. The
sum of the first and last digits is equal to the third digit. Finally, the sum of the second and fourth
digits is twice the sum of the other two digits. What is the third digit of the number?
1. 5
2. 8
3. 1
4. 4
Q. My nephew adores chocolates. He likes biscuits but he hates apples. I told him that he can buy
as many chocolates he wishes but then he must have biscuits twice the number of chocolates and
apples more than biscuits and chocolates together. Each chocolate cost Re. 1. The cost of apple is
twice of the chocolate and four biscuits are worth of one apple. Then which of the following can
be the amount that I spent on that evening on my nephew?
a) Rs. 34
b) Rs. 33
c) Rs. 8
d) None of
these

P, Q, R, S have Rs.290 between them. P has twice as much as R, and Q has three
times as much as S. R and S together have Rs.50 less than P. Find the share of
each.

Q.

A man has a number of one rupee and five-paise coins. He observes that if the
rupees were turned into five-paise coins and the five-paise coins into rupees, he
would have Rs.5.70 more. If the rupees were turned into fifty-paise coins and the
five-paise coins into ten-paise coins, he would have Rs.1.95 less. How much
amount does he have?

A merchant bought certain metres of cloth for Rs.50. He kept 5 m for himself and
sold the rest at Re.1 per metre more than he gave, and made a profit of Rs.10.
How many metres did he buy?

Q Ujakar and Keshav attempted to solve a quadratic equation. Ujakar made a mistake in writing
down the constant term. He ended up with the roots (4,3). Keshav made a mistake in writing
down the coefficient of x. He got the roots as (3,2). What will be the exact roots of the original
quadratic equation?
1. (6,1)
2. (-3, -4)
3. (4,3)
4. (-4, -3)
Q. x and y are real numbers satisfying the conditions 2< x < 3 and -8 < y < -7. Which of the
following expressions will have the least value?
1. x2y
2. xy2
3. 5xy
4. None of
these
DIRECTIONS for questions The batting average (BA) of a test batsman is computed from runs
scored and innings played-completed innings and incomplete innings (not out) in the following
manner:
r1 = number of runs scored in completed innings
n1 = number of completed innings
r2 = number of runs scored in incomplete innings
n2= number of incomplete innings
BA = (r1 + r2)/ n1

To better assess a batsman's accomplishments, the ICC is considering two other measures MBA 1
and MBA2 defined as follows:
MBA1 = r1 / n1 + n2 / n1 + max [0, (r2 / n2 - r1 / n1)]
MBA2 = (r1 + r2 )/ (n1 + n2 )
Q. Based on the information provided which of the following is true?
1. MBA1 BA MBA2
2. BA MBA2 MBA1
3. MBA2 BA MBA1
4. None of these
Q. An experienced cricketer with no incomplete innings has a BA of 50. The next time he bats,
the innings is incomplete and he scores 45 runs. In can be inferred that
1. BA and MBA1 will both increase
2. BA well increase and MBA2 will decrease
3. BA will increase and not enough data is available to assess change in MBA 1 and MBA2
4. None of these

Q.

Find the common root of the equations x2 5x + 6 = 0 and x2 7x + 10 = 0.

Q.
Q.

Solve for x: (3x + 7)/(2x 5) > 3, where x is a whole number.


Solve for x: |x| > 5.

Q.

Solve: |2x + 5| < 9.


.
Solve for x:
(a) |x + 3| + 2 = 17

Q.

(b) |3x 13| + 12 = 7

Q.

Find the value of x: |x + 5| < |2x 7| + 15

Q.

|2x + 4| > |7x 9|

Q. If x > 5 and y < -1, then which of the following statements is true?
1. (x + 4y) > 1
2. x > - 4y
3. - 4x < 5y
of these

4. None

Q. If a, b, c and d are four positive real numbers such that abcd = 1, what is the minimum value
of (l+a) (l+b) (l+c) (l+d)
1. 4
2. 1
3. 16
4. 18
Q

If 3

3x 4
5 and x + y = 4.
8

Find the solution set for y.

If

x
>4, x + y < 0, |x| <3,
y

find the solution set for y.


Q.

Given x, y, z are integers x>y>z. Which is the minimum among the choices?
a. xy(1 z)
b. xz(1 y)
c. yz(1 x)
d. Cannot be determined

In Q.9 if x, y, z < 0 then which of the choices is the largest?


a. xy(1 z)
b. xz(1 y)
c. yz(1 x)
d. Cannot be determined

Q.

Solve for x:
If x2 3x 40 0, x2 25, |x| > 2.

If (x2 + 3x 18) (3x2 7x 6) 0 solve for x.

Q.

The 17th term of an arithmetic progression is 5. If the total number of terms of the
progression is 33 find the sum of all the terms of the progression

Q.

A black monkey is climbing a 200m tall building. It starts from the ground, climbs 4m in
the1st second, slips 2m in the 2nd second, climbs 6m in the 3rd second, then slips 2m in the
fourth, climbs 8m in the 5th second, slips in the next and so on. Find when would it reach
the top of the building?
a. 25th sec
b. 26th sec
c. 27th sec
d. 30th sec

If f(t) = 2t2 + 2/t2 + 5/t + 5t, what is f(1/t)?

If f(x) = xx, find, in terms of f(x), the value of [f(-x)]2

Q.

What is f(2), if f (x) = x3 + x2 + x + 1?

f(0, y + 2) = y + 5
f(x + 5, y) = f (x, y-1) find the value of f (5, 6)

Q f(a, b) = a2 + b3, g (a, b) = a + b Then what is the value of f (3, g (3, 4))
Q

f(x) = x/ (x + 1)
fn (x) = (n 1) fn-1 (x) n > 1
find the value of f4 (1).

Q.

Pakistan is planning to conduct a missile defense test. The path of the missile,
which has to be targeted, is a second-degree polynomial function of time, in
which constant term is negligible. The path of the interceptor, which is supposed
to hit the missile, is also given by the same polynomial, with constant term having
considerable value. Will this test be successful?

If F(x, y, z) = |x| + |y| + |z|, for all real values of x


& G (x, y, z) = |x + y + z|, for all real vales of x, then what will be the relation
between these two functions?

Directions for Q
The following operations are defined for real numbers:
a # b = a + b if a and b both are positive else a # b = 1.
a b = (ab)a + b if ab is positive else a b = 1.
Q. (2 # 1)/(1 2) =
a) 1/8

b) 1

c) 3/8

d) 3

Q. {((1 # 1) # 2) - (101.3 log100.1)}/(1 2) =


a) 3/8
b) 4log100.1/8 c) (4 + 101.3)/8 d) None of these
Q. ((X # - Y)/(- X Y) = 3/8, then which of the following must be true?
a) X = 2, Y = 1 b) x > 0, Y < 0 c) X, Y both positive d) X, Y both negative
Directions for Q. : These questions are based on the situation given below:
Let x and y be real numbers and let
f(x,y) = |x+ y|, F(f(x,y)) = - f(x,y) and G(f(x,y)) = - F(f(x,y))
Q. Which of the following statement is true?
a) F(f(x,y)) . G(f(x, y)) = - F(f(x, y)). G(f(x, y))
b) F(f(x, y)). G(f(x, y)) > - F(f(x, y)). G(f(x, y))
c) F(f(x, y)). G(f(x, y)) G(f(x, y)) . F(f(x, y))
d) F(f(x, y) + G(f(x, y)) + f(x, y) = f(- x, - y)
Q. What is the value of f(G(f(1, 0)), f(F(f(1 2)) , G(f(1, 2))))?
a) 3
b) 2
c) 1
d) 0
Q. Which of the following expressions yields x2 as its result?
a) F(f(x, - x) . G(f(x, - x)
b) F(f(x, x)) . G(f(x, x)). 4
c) - F(f(x, x) . G(f(x, x) log2 16
d) f(x, x). f(x, x)
Directions for Q.
For three distinct real numbers x, y and z, let
f(x, y, z) = min (max(x, y), max (y, z), max (z, x))
g(x, y, z) = max (min(x, y), min (y, z), max (z, x))
h(x, y, z) = max (max(x, y), max(y, z), max (z, x))
j(x, y, z) = min (min (x, y), min(y, z), min (z, x))
m(x, y, z) = max (x, y, z)
n(x, y, z) = min (x, y, z)

Q. Which of the following is necessarily greater than 1?


a) (h(x, y, z) - f(x, y, z)) / j(x, y, z)
b) j(x, y, z)/h(x, y, z)
c) f(x, y, z)/g(x, y, z)
d) (f(x, y, z) + h(x, y, z) - g(x, y, z))/j(x, y, z)
Q. Which of the following expressions is necessarily equal to 1?
a) (f(x, y, z) - m(x, y, z))/(g(x, y, z) - h(x, y, z))
b) (m(x, y, z) - f(x, y, z))/(g(x, y, z) - n(x, y, z))
c) (j(x, y, z) - g(x, y, z)) /h(x, y, z)
d) (f(x, y, z) - h(x, y, z) /f(x, y, z)
Q Which of the following expressions is indeterminate?
a) (f(x, y, z) - h(x, y, z))/(g(x, y, z) - j(x, y, z))
b) (f(x, y, z) + h(x, y, z) + g(x, y, z) + j(x, y, z))/(j(x, y, z) + h(x, y, z) - m(x, y, z) - n(x, y,
z))
c) (g(x, y, z) - j(x, y, z))/(f(x, y, z) - h(x, y, z))
d) (h(x, y, z) - f(x, y, z))/(n(x, y, z) - g(x, y, z))

If x is positive what is the greatest value of (5-x) (x + 3)?.

Find all values of x which satisfy x2 + 6x 27 > 0 and 4 + 3x x2 >


Solve the inequality 2x2 + 9x 35 > 0.

Solve the inequality 3x2 20x + 17 0.

Solve the inequality |x/2| < 2.

Let y = min {(x + 6), (4 x)}. If x R, what is the maximum value of y?

Directions for Q.: These questions are based on the situation given below:
In each of the questions 89 to 92 a pair of graphs F(x) and F1(x) is given. These are composed of
straight-line segments, shown as solid lines, in the domain x (- 2, 2).
If F1(x) = - F(x) choose the answer as a;
If F1(x) = F(- x) choose the answer as b;
If F1(x) = - F(- x) choose the answer as c;
And if none of the above is true, choose the answer as d.

F(X)

F1(X)

-2

-2

2 X

-2
b) b

a) a
90.

-2

-2
c) c

d) d

F(x)

F1(x)

-2

a) a

-2
b) b

-2
c) c

d) d

91.

F(x)
2

F1 (x)
2

-2

-2

c) c

d) d

-2
b) b

a) a

92.

-2

F(x)

F1(x)

1
1
-2

1
1

-2

x
-1

-1
-2

-2
a) a

b) b

c) c

d) d

You might also like